Gas right now is $ 2.99 per gallon !! AH !!! How much will it clllost to fill the tank of my car that holds 9 gallons ?

Show all work...​

Answers

Answer 1

Answer:

$26.91

Step-by-step explanation:

Multiply $2.99 with 9.

Reason is that its $2.99 a gallon for gas and you need 9 gallons to fill up your vehicle

That equals $26.91.

Answer 2

Answer:

$26.91

Step-by-step explanation:

Since 1 gallon costs $2.99, multiply 2.99 by 9.

2.99 x 9

= $26.91

It will cost you $26.91 to fill up your tank that holds 9 gallons.


Related Questions

How long should an escalator be if it is to make an angle 30 with the floor and

Answers

Answer:

it should be 23 feet long

Step-by-step explanation:

which transformation would result in a geometric figure

Answers

Answer:

You reflect it, Translated and Rotate it

Step-by-step explanation:

hope this helps

Find the slope of the line graphed below.

Answers

Slope is the change in y over the change in x.

Using the red dots:

Slope = (4 -1)/(4- -1) = 3/5

Slope = 3/5

Which of the following equations represents an ellipse having vertices
located at (2,9) and (2,-5) and foci located at (2,5) and (2,-1)?

Answers

Answer:

(x-2)^2/40 + (y-2)^2/49 = 1

Step-by-step explanation:

Brainlisted for right answer!!! 25 points!!! Please help me!!! I really need this done!!!

Answers

r = 23(4) = 92
h = 23-7 = 16

What is 300251 rounded to the nearest Highest Place

Answers

300000 , make me the barinliest

Answer:

30,000

Step-by-step explanation:

Alex bought 5 yards of fabric to make pillows. What is the length of the fabric in inches?
1 foot = 12 inches
1 yard = 3 feet
1 mile = 5,280 feet
1 mile = 1,760 yards

Answers

5 yards is 180 inches

Answer:

180 inches

Step-by-step explanation:

Convert 5 yards to feet. According to the information given, 1 yard = 3 feet. So now we have to multiply 5 yards by 3 feet which gives you 15 feet. Secondly, convert 15 feet to inches. It stated that 1 foot = 12 inches, so we have to do 15 feet multiplied by 12 inches to give you, 180 inches. So therefore, the length of the fabric is 180 inches.

i dont understand if anybody can help me i would be happy​

Answers

Answer:

(2, 3)

Step-by-step explanation:

The rule for this dilation is:

(x, y) → (x/2, y/2)

Apply this rule to point B(4, 6):

B(4, 6) → B'(2, 3)

Simplify completely: - 30+ (5) – (s) Pay attention to
the negatives!

Answers

Answer:1

Add the numbers

3

0

+

5

30+5−s

3

5

35−s

2

Rearrange terms

Solution

+

3

5

Step-by-step explanation:

Use the following experiment. A state lottery game consists of choosing one card from each of the four suits in a standard deck of playing cards. (There are 13 cards in each suit.)

Count the number of ways in which four 8's can be chosen.

Answers

Answer:

24

Step-by-step explanation:

there are 4 way of choosing an ace card after choosing an ace card now we have 3 ways of choosing a king after choosing a king there are two ways of choosing a queen after choosing queen there is only one way of choosing a jack .

so the number of elements in the event is =4×3×2×1=24

we use permutation here for finding the total number of elements

Using the concept of combination, since the order of selection doesn't matter, the number of ways in which four 8's can be chosen is 1

There are 13 cards in a suit :

Number of cards numbered 8 in a suit = 1

Number of ways of choosing a specific card :

(13C1) ÷ 13 = (13 ÷ 13) = 1 way

Since there are four different suits :

The number of ways of selecting a card numbered 8 from 3 each suit will be 1.

Hence, the number of ways in which four 8's can be chosen would be :

1 × 1 × 1 × 1 = 1

Therefore, there is only 1 way of choosing four 8's

Learn more : https://brainly.com/question/24555248

what's 5.1 x 10^0 in standard notation? plz I meed it for my math class ​

Answers

Answer:

5.1

Step-by-step explanation:

Since any (nonzero) number to the zero power is equal to one, 10^0 = 1

5.1*1 = 5.1

prove that the sum of any 5 consecutive natural numbers is divisible by 5​

Answers

Answer:

Let the 5 consecutive numbers be x, x + 1, x + 2, x + 3 and x + 4.

The sum of the 5 consecutive numbers,

x + x + 1 + x + 2 + x + 3 + x + 4

= x + x + x + x + x + 1 + 2 + 3 + 4

= 5x + 10

To prove : -

The sum of any 5 consecutive natural numbers is divisible by 5​.

Proof : -

( 5x + 10 ) / 5

= ( 5x / 5 ) + ( 10 / 5 )

= x + 2

Hence proved.

Therefore,

The sum of any 5 consecutive natural numbers is divisible by 5​.

Jdjsjdjdnndnxnxnxnxjxnnxnxnxnxncncncnxnxnnxnxnxncnccncnncncncncjcjdnx.

Answers

Answer:

?

Step-by-step explanation:

Answer: so 2 +5/6 =2

Step-by-step explanation:

The reson is I’m doing this for a challenge ty

Please someone help me.. ASAP ​

Answers

Answer:

1c2a3b

Step-by-step explanation:

After 5 years, Ben’s income has undergone several increases. Last month, Ben put $600 in savings, which was 10% of his net income. What is Ben’s new monthly net income?

Answers

Answer:

$6,000

Step-by-step explanation:

10/100 = x/600

= (600 * 100)/10

= 600 * 10

Anyone know how I can solve RL from this equation, because I have been trying and I can't get the result[tex]10=-2,5+\frac{0.02*RL+2,5}{625} (625+500)[/tex]
* this sign is to multiply
expected result is = 222.2

Answers

Answer:

steps below

Step-by-step explanation:

10 = -2.5 + ((0.02*RL + 2.5) / 625)*(625+500)

0.02*RL + 2.5 = (10+2.5)*625 / (625+500)

                         = 12.5*625 / 1125

                         =6.9444444

RL = (6.9444444 - 2.5) / 0.02

     = 4.44444... / 0.02

     = 222.22222 ....

Janet hiked 3/8 mile in 1/4 hour how fast did she hike in miels per hour

Answers

Answer:

[tex]1\frac{1}{2}[/tex] miles per hour

Step-by-step explanation:

To solve, multiply both sides by 4;

[tex]\frac{1}{4} *4=4\\\\\\[/tex]

[tex]\frac{3}8} *4= \frac{12}{8}[/tex] or [tex]1\frac{1}{2}[/tex]

Hope this helps! :)

Hi, what is this number in expanded form, please let me know?

Answers

Answer:

5,078.401

Step-by-step explanation:

5,000 + 70 + 8 + 0.4 + 0.001 = 5,078.401

25.638= Twenty five and six hundred thirty eight how would this be written out And how to find this Question to the answer ​

Answers

Answer:

25.638

Step-by-step explanation:

because I am smart

In the figure shown, help

Answers

9514 1404 393

Answer:

  the correct answer is marked

Step-by-step explanation:

Given that only lines a and b are parallel, any triangle not involving lines a or b, or not having K as a vertex, will not be similar to ΔJKH.

Please help fast!! Quarter ends tonight!

Answers

Answer:

-18

Step-by-step explanation:

2 x -2^3 + 3 x 2 x -1 - 4 x -1^2 = -18

Answer I think is 18

Step-by-step explanation:

The first part is 16, then the second part is 6, then the third part is 4. So then it would be 16+6-4=18, if I'm not mistaken.

Kevin and Joaquim had a total of $620. Joaquim had $100 more than Kevin. How much money did Kevin have?

What is the answer for this question?

Answers

Answer:

$260

Step-by-step explanation:

620-100=520 ( to make both of them equal)

520÷2=$260

Hope this helps! Thanks.

what is a rapid loan?

Answers

Answer:Rapid Loans Fast is the premier provider of cash advances, emergency and weekend loans. ... Thanks to the introduction of modern IT, the application for an online loan will take you no more than 5-10 minutes. Approval occurs automatically, after which the payday loans are instantly transferred to the borrower's bank card.

Step-by-step explanation:

Write the equation for the function graphed below.

Answers

Given the vertex, (2, 7), and that it is a downward-facing parabola:

We can use the vertex form of the quadratic function:

f(x) = a(x - h)^2 + k
where:

Vertex = (h, k)

“a” determines whether the graph opens up or down.

* If “a” is negative, the graph opens down.

The value of “a” also makes the parent function wider or narrower.

The value of “h” determines how far left or right the parent function is translated.

The value of “k” determines how far up or down the parent function is translated.

Given these information, we can plug in the value of the vertex into the vertex form:

f(x) = a(x - h)^2 + k

f(x) = a(x - 2)^2 + 7

Next, to solve for “a”, we can use one of the points on the graph, (1, 5), and plug these values into the equation:

f(x) = a(x - 2)^2 + 7

5 = a(1 - 2)^2 + 7

5 = a(-1)^2 + 7

5 = a(1) + 7

Subtract 7 from both sides:

5 - 7 = a(1) + 7 - 7
-2 = a

Therefore, a = -2. Now, we can establish the following quadratic function in vertex form:

f(x) = -2(x - 2)^2 + 7


Please mark my answers as the Brainliest if you find my explanation helpful :)

Which is longer
1m 200 cm 3ft 700mm 1 yard

Answers

Answer:

200 centimeters is the longest of the five.

Apply the distributive property to factor out the greatest common factor 21+25

Answers

Answer:

there is not a common factor

Step-by-step explanation:

Use the Venn diagram to determine which sets each number belongs to.
a. 18
b. -17
c. -23
d. ⅜

Answers

Answer:

a. whole number, integer, rational

b. integer, rational

c. integer, rational

d. rational

Step-by-step explanation:

hope it helps

Please help!!

2y = -1 + 9
3х — бу :
-15
M

Answers

• Rearrange first equation
x + 2y = 9

• Multiply the first equation by 3
3(x + 2y = 9)

• Rewrite equation 1
3x + 6y = 27

• Write both equations
3x + 6y = 27
3x - 6y = -15

• Subtract equation 1 from equation 2
3x + 6y = 27
-
3x - 6y = -15

• Write the reaming values
12y = 42

• Divide both side by 12 to solve for y
12y = 42 ➗ 12

• Write value of y
y = 3.5

• Substitute the value of y into one of the original equations
2(3.5) = -x + 9

• Use algebra and sole for x
7 = -x + 9
x = 9 - 7
x = 2

• Write final point
(3.5, 2)

Let A and B be two disjoint events such that P(A) = 0.47 and P(B) = 0.3.

What is P(A or B)?

Answers

Answer:  0.77

Work Shown:

P(A or B) = P(A) + P(B)

P(A or B) = 0.47 + 0.30

P(A or B) = 0.77

Notes:

The formula only works if A and B are disjoint, aka mutually exclusiveIf A and B are not disjoint, then use this formula: P(A or B) = P(A) + P(B) - P(A and B)

Answer:

P(A or B) = 0.77

Step-by-step explanation:

may give brainliest
please answer

Answers

Answer:

See below

Step-by-step explanation:

The points are:

9/2 = 4.5 is in between 4 and 5-7/2 = -3.5 is in between -4 and -3

See attached

9/2=[tex]4\dfrac{1}{2}[/tex]-7/2=[tex]-3\dfrac{1}{2}[/tex]

Refer to the attachment

Other Questions
Please answer number 19 please I beg you! Given the sequence 1/2 ; 4 ; 1/4 ; 7 ; 1/8 ; 10;.. calculate the sum of 50 terms Answer the question below thx what hard ships did people face in the great depression plz hurry My mother is a few years_ my father Can we eat to starve cancer? Theoretical yield of NaHCO3 = Na + H2O + CO + CO2 Carbonic anhydrase is an enzyme that converts carbon dioxide intobicarbonate so red blood cells can transport it. What is the best way toincrease the rate of reaction?A-Add more carbonic anhydraseB-Add more bicarbonateC-Reduce the amount of waterD-Reduce the amount of carbon dioxide NO LINKS!!! 1. What is the structure of a proof in geometry?2. Is ray AB the same as ray BA?3. What is the difference between the m Do the work will give brainlist What number is represented by 4x2+17x+2if x=10? Before the Buffalo Woman is killed by the hunters from Do, she: _________a) Tells them how to slay her and asks them to take Sogolon as their reward for killing her. b) Claims that Maghan Kon Fatta is her brother and promises them great rewards if they do not kill her.c) Attacks and kills their families when the families are working in the fields harvesting crops. d) Accuses the hunters of betraying their clan and warns them that they will not survive the hunt. 0+1+2+3+4+5+6+7+8+9+10+11+12+13+14+15+16+17+18+19+20 b) f(x) = 3x2 5x + 1, (1, 7), Write an algebraic expression for each situation below. Evaluate the expression for the given values. Jean earns a base salary of $500.00 per week as a sales consultant at retail store. She also earns 12% of all sales she makes. How much does she make in a week when she sells $650.00 worth of inventory? Make the following plural:el profesor QUESTION 5A. Aluminum carbonate(03.04 LC)Match the formula to the correct name. Al2O3e Al(HCO3)3Al2(CO3)3B. Aluminum oxideC. Aluminum bicarbonateQUESTION A PLEASE HELP I REALLY DONT UNDERSTAND THANK YOUU So I have this question I cannot figure out 3w+w2+1=10w what is the the answer